Search found 101 matches


my study startegy..... well, I started out with Kaplan, learning the types of questions and how to approach them. then I started answering quizes from Kaplan and from the official guides. then I came upon beat the gmat, I must say it really helped, because i tried to answer all the questions and pos...

by agps

Fri Oct 12, 2007 1:40 pm
Forum: I just Beat The GMAT!
Topic: 710- good enough for me
Replies: 5
Views: 3045

actually it's easier than that. the slope is given by the equation (y2-y1)/(x2-x1).

in this case
(-1, -7) and (2, 5)

(5-(-7))/(2-(-1)) = 12/3 = 4
or
((-7)-5)/((-1)-2) = -12/-3 = 4

by agps

Thu Oct 04, 2007 10:05 am
Forum: Data Sufficiency
Topic: Set 6 Q27
Replies: 5
Views: 1784

710- good enough for me

I took it, and beat it 700+ was my goal and I got it.

during the test i thought it was going really bad, all i saw were questions i thought were too difficult, but then i remembered, maybe this is a good thing. turned out to be :P

good luck everyone. beat it

by agps

Thu Oct 04, 2007 12:15 am
Forum: I just Beat The GMAT!
Topic: 710- good enough for me
Replies: 5
Views: 3045

the trick is that you can't have 3/4 children or men on the trip, it must be an integer. 1) tells you that the number of children must be a multiple of 5, actually, must be a multiple of 10, because multiples of 5 ending in 5 are not multiples of 2 (2 children for each 5 women) so, 10, 20, 30, all a...

by agps

Sun Sep 23, 2007 9:28 am
Forum: Problem Solving
Topic: GMAT Prep
Replies: 4
Views: 1730

answer is D. as pointed out by kajcha, I is wrong and it's the onbly answer without I in it.

with the correction made by radhika1306 we can confirm that III is also true (-1)^0 = 1 = -k.

by agps

Sat Sep 15, 2007 9:52 am
Forum: Problem Solving
Topic: set 26 q 32
Replies: 3
Views: 1689

answer should be D.

the sets are small enough, just write down the possibilities.
let the employees be a,b and c
office 1 - office 2
--------------------
abc -
ab - c
ac - b
cb - a
a - bc
b - ac
c - ba
- abc

by agps

Tue Sep 11, 2007 9:44 am
Forum: Problem Solving
Topic: Scoretop 5 - Q6
Replies: 2
Views: 1527

good point samir. I have to be more caerefull, DS is always a trap if we don't look caerfully

by agps

Sat Sep 08, 2007 8:38 am
Forum: Data Sufficiency
Topic: GMAT Prep
Replies: 4
Views: 2497

answer is C. from the question let m be Mary's annual salary, j Jim's and k Kate's. m-j=2(m-k) m>j and m>k from 1) j=30k, can't figure out m or k, insufficient from 2)k=40k, can't figure out m or j, insufficient together we know that m>k>j m-30=2(m-40) m-30=2m-80 m=50 so the mean is 50+40+30/3 = 120...

by agps

Sat Sep 08, 2007 2:43 am
Forum: Data Sufficiency
Topic: GMAT Prep
Replies: 4
Views: 2497

from 1) x and y are either both positive and x>y or x and y have different signal and x> 0. more than 1 possible answer, insufficient. from2) both x and y have the same signal negative or positive we don't know, insufficient combining both, from 1) x is always positive and from 2) x and y both have ...

by agps

Sat Sep 08, 2007 2:37 am
Forum: Data Sufficiency
Topic: GMAT Prep
Replies: 7
Views: 2466

we all agree that the equation simplifies to z =x*z if x=1 z=z and z can be any value, not necessairly 0 if x=0 z must be 0, or else the equation is false. try out in the original equation if x=1 XY+Z= X(Y+Z) results in y+z=y+z 0 = 0, regardless of the value of Z so E can't be right, even if z were ...

by agps

Thu Sep 06, 2007 2:10 am
Forum: Problem Solving
Topic: Can anyone solve this question from GMAT Prep?
Replies: 8
Views: 2146

the triangle to the left is a 3-4-5 special triangle. the triangle to the right is a 5-12-13 special triangle. since this is a isosceles triangle (can't be equilateral because of the angle 55º), the other angle that is not x is also 55º. sum of angles of triangle is 180, so x=180-55-55 = 70. this re...

by agps

Wed Sep 05, 2007 11:21 pm
Forum: Problem Solving
Topic: Gmat Prep 2- Triangle Question
Replies: 2
Views: 1588

I'm actually using it, just not calling it permutations.... "odd numbers end in 1,3,5,7,9. so 5 possibilities" this is 5P1 = 5!/4! = 5 "8 possibilites left for the 2nd digit (can't be 8 or the units digit)" this is 8P1 = 8!/7! = 8 "(for 9) the same applies, but the units dig...

by agps

Wed Sep 05, 2007 4:20 pm
Forum: Problem Solving
Topic: Permutation
Replies: 11
Views: 2346

"So X + 4( 1.1X) = 132000

so 12X =132000 "
I believe you mean
So X + 10( 1.1X) = 132000
so 12X =132000

good way to solve it by the way. 8)

by agps

Wed Sep 05, 2007 4:16 pm
Forum: Problem Solving
Topic: gmat prep
Replies: 5
Views: 1586

the calculation 4/8*3/7*2/6 is the probability that the 1st, 2nd and 3rd choices are sports magazines. = 24/336 = 1/14

now you want all other options except this one. so 1-1/14 is the answer or 13/14

by agps

Wed Sep 05, 2007 3:48 pm
Forum: Problem Solving
Topic: GMAT Prep test 2
Replies: 5
Views: 1876

my mistake was to consider the 15 min before Paul left the gas station, those don't count. :lol:

by agps

Wed Sep 05, 2007 3:43 pm
Forum: Problem Solving
Topic: set 24 q 36
Replies: 9
Views: 2185